key: cord-018363-qr1pk78u authors: Casey, Ashley; Conrad, Kevin title: Consultative and Comanagement date: 2015-10-10 journal: Absolute Hospital Medicine Review DOI: 10.1007/978-3-319-23748-0_2 sha: doc_id: 18363 cord_uid: qr1pk78u This chapter covers the role of the hospitalist as a consultant and their interaction with surgical specialties. Included are discussions of perioperative care in the hospital and clinic. Oral and parenteral nutrition for the hospitalized patient are examined. A special emphasis is placed on palliative care for the hospitalized patient. Comanagement of surgical patients with an emphasis on orthopedics is reviewed. A 66-year-old male presents to the emergency room with a chief complaint of a severe headache that developed approximately 10 h ago. He describes the headache as the worst headache of his life. He has a history of myelodysplasia for which he has been followed as an outpatient. He reports no history of spontaneous bleeds and denies any spontaneous bruising. On physical examination, he is alert and oriented, and his speech is slightly slurred. The prothrombin time and activated partial thromboplastin time are within normal range. A CT scan is performed in the emergency room that shows an intracerebral bleed with a mild amount of extravasation of blood into the ventricular system. Which of the following is the most appropriate minimum platelet threshold for this patient? A) 40,000 B) 60,000 C) 100,000 D) 150,000 Answer: C Thresholds for platelet transfusions are undergoing close examination. Some areas continue to provoke debate especially concerning the use of prophylactic platelet transfusions for the prevention of thrombocytopenic bleeding. Guidelines recommend maintaining platelet count at 100,000 after a central nervous system bleeding event. This would also be the case immediately prior to and after surgery performed on the central nervous system. This patient has a potentially life-threatening intracranial bleeding. The bleeding source is probably secondary to hypertensive disease and not thrombocytopenia. However, the patient is at continued risk for extension of the intracerebral bleeding because of her thrombocytopenia. Guidelines do not suggest additional benefi ts to maintaining platelet counts >100,000. A 44-year-old woman undergoes preoperative evaluation prior to surgery to repair a congenital defect of her pelvis. Her expected blood loss is 2.0 l. She has a prior history of severe anaphylactic reaction to a prior erythrocyte transfusion that she received for postpartum hemorrhage at age of 27 years. In addition she has a history of rheumatoid arthritis. On physical examination, the temperature is 36.8 °C (98.5 °F), blood pressure is 140/70 mmHg, and heart rate is 76 bpm. Laboratory studies indicate a hemoglobin level of 12.0 g/dL, a leukocyte count of 6500 μL, and a platelet count of 150,000 μL. Previous laboratory studies indicate an IgG level of 800 mg/dL and an IgM level of 65 mg/dL. Which of the following is the most appropriate erythrocyte transfusion product for this patient? A) Leuko-reduced blood B) Cytomegalovirus-negative blood C) Irradiated blood D) Phenotypically matched blood E) Washed blood Answer: E This patient has IGA defi ciency. The most appropriate product to minimize the risk of an anaphylactic transfusion reaction in this case is washed erythrocytes. Most patients with an IgA defi ciency are asymptomatic. They are prone to gastrointestinal infections such as giardia. They also have an increased risk of autoimmune disorders such as rheumatoid arthritis and systemic lupus erythematosus. Some patients with IgA defi ciency have anaphylactic reactions to blood products containing IgA. Fresh frozen plasma (FFP) is the main blood component containing IgA antibodies. Anaphylaxis may occur with a variety of transfusions including FFP, platelets, and erythrocytes. Washing erythrocytes and platelets removes plasma proteins and greatly decreases the incidence of anaphylaxis. A 34-year-old man with a history of superfi cial thrombophlebitis presents with bilateral foot pain of 3-days duration. Over the 6 months, he has had several distinct episodes of severe burning pain of the foot and several toes. The pain persists at rest and is debilitating. The patient smokes one to two packs of cigarettes a day. On physical examination, he is thin; his feet are erythematous and cold. There are ulcerations noted distally on both feet. The femoral pulses are strong and intact, and the dorsalis pedis and posterior tibialis pulses are absent bilaterally. No discoloration is noted on his leg and a normal hair pattern is noted on his legs. The pain is not worsened by deep palpation. What is the most likely diagnosis for this patient? A) Plantar fasciitis B) Spinal stenosis C) Thromboangiitis obliterans D) Raynaud phenomenon E) Atherosclerotic claudication Answer: C This patient has thromboangiitis obliterans, also called Buerger's disease. This results from infl ammatory blockage of arterioles in the distal extremities and is usually seen in male smokers who are typically less than 40 years of age. Other typical features include a history of recurrent thrombophlebitis and rest pain. Distal pulses are often absent. Plantar fasciitis is usually relieved with rest. Weight bearing and exercise exacerbate it. Spinal stenosis usually occurs in older patients. It is exacerbated by standing or walking and is relieved by rest. Atherosclerotic claudication is also seen in older patients. It has a steady progression. It starts with exercise-related pain and progresses slowly to pain at rest. Raynaud phenomenon is seen mostly in women. It is caused by vasospasm of small arterioles. It more commonly occurs in the hands but can be seen in the feet. The vasospasm is precipitated by cold, temperature change, or stress. Color changes, which can be profound, occur in the digits from white to blue to red. Pain is usually not severe and peripheral pulses remain intact even during episodes of vasospasm. In Buerger's disease, among patients who stop smoking, 94 % avoid amputation. In contrast, among patients who continue using tobacco, there is an 8-year amputation rate of 43 %. Espinoza LR. Buerger's disease: thromboangiitis obliterans 100 years after the initial description. Am J Med Sci. 2009;337(4):285-6. Olin JW, Young JR, Graor RA, Ruschhaupt WF, Bartholomew JR. The changing clinical spectrum of thromboangiitis obliterans (Buerger's disease). Circulation. 1990; 82 (5 Suppl) : . Preoperative malnutrition is associated with which outcome in patients undergoing gastrointestinal surgery? A) Increased 30-day mortality B) Increased 60-day mortality C) Increased length of stay D) All of the above Answer: D Good nutritional status is an important factor in the outcome of gastrointestinal surgery. Several studies have confi rmed this. Preoperative malnutrition is an independent predictor of length of hospital stay, 30-day, and 60-day mortality, as well as minor medical complications, in patients undergoing gastrointestinal surgery. Preoperative nutrition including total parenteral has been proven to be benefi cial in malnourished patients undergoing gastrointestinal surgery. Reference Burden S, Todd C, Hill J, Lal S. Pre-operative nutrition support in patients undergoing gastrointestinal surgery. Cochrane Database Syst Rev. 2012;(11):CD008879. A 52 year-old male presents with the chief complaint of daily seizures. He reports that he has had seizures weekly for the past several years since an automobile accident, but these have increased to nearly daily in the past few weeks. He states he takes levetiracetam, but is not certain of the dose. While in the emergency room, he has a generalized grand mal seizure and is given lorazepam. He has recently moved to the area and has no old records. He is admitted to the hospital medicine service and a 24 h EEG is instituted. On the fi rst night of his admission, he has an apparent seizure but no seizure activity is noted on the EEG. The next morning he develops an inability to move the left side of his body and dysar-thria. Urgent MRI of his head reveals no evidence of acute cerebrovascular accident. The most likely cause of his paralysis is? A) Early cerebral infarction B) Todd's paralysis C) Malingering D) Migraine variant E) Conversion disorder Answer: C This patient has several factors that suggest malingering. He presents with two relatively easy to mimic symptoms. First, he has a seizure with no eleptiform activity and then paralysis with a normal MRI. His recent travel from another area is also suggestive of the diagnosis. Malingering is not considered a mental illness and its diagnosis and treatment can be diffi cult. Direct confrontation may not work best. Hostility, lawsuits, and occasionally violence may result. It may be best to confront the person indirectly by remarking that the objective fi ndings do not meet the objective criteria for diagnosis. It is important to demonstrate to the patient that his abnormal behavior has been observed and will be documented. At the same time an attempt should be made to allow the patient who is malingering the opportunity to save face. Obviously this can be a challenge. Invasive diagnostic maneuvers, consultations, and prolonged hospitalizations often do more harm than good and add fuel to the fi re. People who malinger rarely accept psychiatric referral, and the success of such consultations is minimal. It may be considered to address a specifi c psychiatric complaint. The most common goals of people who malinger in the emergency department are obtaining drugs and shelter. It may be benefi cial to offer the patient some limited assistance in these areas. In the clinic or offi ce, the most common goal is fi nancial compensation. 259. A 60-year-old male with chronic obstructive pulmonary disease is admitted for a hip fracture sustained after a fall. He undergoes surgery without complication. On the second day of hospitalization, he develops some mild dyspnea and nonproductive cough. He is currently on 2 l of oxygen at home and states that he will often get somewhat short of breath with any change in his living situation. On physical exam, the patient appears comfortable. His temperature is 37.8 °C (100.1 °F), heart rate is 70 bpm, and respirations are 16 per minute. Oxygen saturation is 96 % on pulse oximetry with 2 l. A chest X-ray shows no acute changes and white blood cell count is within normal limits. Which of the following is the appropriate management of this patient? A) Prednisone B) Doxycycline plus prednisone C) Levofl oxacin D) Azithromycin Answer: A American College of Chest Physician guidelines for chronic obstructive pulmonary disease exacerbation support inhaled beta agonists and steroids alone for mild fl ares. In this particular case, the patient is having a mild exacerbation of his typical chronic obstructive pulmonary disease. Antibiotics should be reserved for moderate to severe cases. The criteria for moderate disease exacerbation include cough, change in color of sputum, and increased shortness of breath. 260. A 74-year-old man is admitted for cough, dyspnea, and altered mental status. The patient is noted to be minimally responsive on arrival. Results of physical examination are as follows: temperature, 38.9 °C (102.1 °F); heart rate, 116 bpm; blood pressure, 96/60 mmHg; respiratory rate, 35 breaths/min; and O 2 saturation, 74 % on 100 % O 2 with a nonrebreather mask. The patient is intubated urgently and placed on mechanical ventilation. On physical exam, coarse rhonchi are noted bilaterally. A portable chest X-ray reveals good placement of the endotracheal tube and lobar consolidation of the right lower lobe. Empirical broad-spectrum antimicrobial therapy is started. Which is true concerning his nutritional management? A) Enteral nutrition is less likely to cause infection than parenteral nutrition. B) Parenteral nutrition has not consistently been shown to result in a decrease in mortality, compared with standard care. C) The use of oral supplements in all hospitalized elderly patients has been shown to be benefi cial. D) Immune-modulating supplements are no better than standard high-protein formulas in critically ill patients. E) All of the above Answer: E Comparisons of enteral nutrition with parenteral nutrition have consistently shown fewer infectious complications with enteral nutrition. Several studies have looked at specialized feeding formulas in the treatment of the critically ill. There is little evidence to support their use over standard high-protein formulas. In one study among adult patients breathing with the aid of mechanical ventilation in the ICU, immune-modulating formulas compared with a standard high-protein formula did not improve infectious complications or other clinical end points. Elderly patients require special consideration. A trial in 501 hospitalized elderly patients randomized to oral supplements or a regular diet showed that, irrespective of their initial nutritional status, the patients receiving oral supplements had lower mortality, better mobility, and a shorter hospital stay. 261. You are called to see a patient urgently in the postpartum ward. She is a 32-year-old female who, 20 min prior, had an uneventful vaginal delivery. In the past 20 min, the patient has become abruptly short of breath, hypoxic, and severely hypotensive with a blood pressure of 72/palpation mm Hg. On physical exam, she is obtunded and in serve respiratory distress. She has no signifi cant past medical history documented and has had an uneventful pregnancy. Mild wheezes with decreased breath sounds are heard. Chest radiograph and arterial blood gasses are pending. The most likely diagnosis is? A) Pulmonary embolism B) Sepsis C) Peripartum cardiomyopathy D) Amniotic fl uid embolism E) Eclampsia Answer: D Amniotic fl uid embolism is a rare complication of pregnancy. It presents acutely during and immediately after delivery, usually within 30 min. The exact mechanisms are unclear, but it is thought that amniotic fl uid gains entry into the maternal circulation. This triggers an intensive infl amma-tory reaction, resulting in pulmonary vasoconstriction, pulmonary capillary leak, and myocardial depression. Patients present with acute hypoxemia, hypotension, and decreased mental status. Treatment is supportive but may be improved by early recognition and cardiopulmonary resuscitation. The other answers do occur in pregnancy, but the severity, rapid onset, and timing to delivery strongly suggest amniotic fl uid embolism. The mortality rate may exceed 60 %. Immediate transfer to an intensive care unit with cardiovascular resuscitation is recommended. Over the past 4 days since surgery, she has been on parenteral nutrition. Oral intake has been started gradually 2 days ago. Diarrhea has occurred both at night and day. Stool cultures and Clostridium diffi cile polymerase chain reaction are negative. Her current medications include low-molecular-weight heparin as well as loperamide two times daily. Which of the following is the most appropriate management? A) Increase loperamide. B) Initiate cholestyramine. C) Initiate omeprazole. D) Stop oral intake. E) Decrease lipids in parenteral nutrition. Answer: C Patients who have undergone signifi cant bowel resection should receive acid suppression in the postoperative period with a proton pump inhibitor. This patient has short-bowel syndrome. Any process that leaves less than 200 cm of viable small bowel or a loss of 50 % or more of the small intestine as compared to baseline places the patient at risk for developing shortbowel syndrome. In short-bowel syndrome, there is an increase in gastric acids in the postoperative period. This can lead to inactivation of pancreatic lipase, resulting in signifi cant diarrhea. Stopping the patient's oral intake may lead to temporary improvement. It is important that the patient continues her oral feedings, as this will eventually allow the gut to adapt and hopefully resume normal function. A 58-year-old female who underwent an elective cholecystectomy is noted to be in atrial fi brillation by telemetry. Her heart rate is 108 bpm. She has a history of hypertension. Her medications are verapamil and fullstrength aspirin. She states that several years ago, she had palpitations after exercise, but that has since resolved, and she has noticed no problems. You are consulted by the surgical team for management of her heart rate in preparing her for discharge. On physical exam she appears in no distress and is not short of breath. Which of the following is the appropriate management of the patient's atrial fi brillation? A) Maintain her current dose of verapamil. B) Increase her dose of verapamil with a target rate of 80 beats per minute. C) Add digoxin to control her heart rate to a target of 80 beats per minute. D) Consult cardiology for possible cardioversion. Answer: A A 2010 study compared lenient control of heart rate less than 110 beats per minute to more strict control of less than 80 beats per minute. The study found that achieving strict heart rate control resulted in multiple admissions with no perceivable benefi t outcomes. In this particular case, a heart rate of 108 bpm is acceptable, and patient the can be discharged on her current medications. Follow-up with her primary care physician should be obtained to monitor heart rate. Digoxin can be used in the acute setting but does little to control the ventricular rate in active patients. It is rarely used as monotherapy. Caution should be exercised in elderly patients with renal failure due to toxicity. Digoxin is indicated in patients with heart failure and reduced LV function. 266. You are called to the fl oor to see a patient who has developed acute onset of shortness of breath. She is a 56-year-old female who was admitted for upper GI bleed. She is currently receiving her fi rst unit of packed erythrocytes, which was started 1.5 h ago. On physical examination, temperature is 38.9 °C(102 °F), blood pressure is 110/65, pulse rate is 115 beats per minute, and respirations are 22 per minute. Her current oxygenation is 83 %. She has been placed on 3 l by nasal cannula. No peripheral edema is noted. Mild wheezes and diffuse crackles are heard throughout her lung fi elds. A stat X-ray is ordered which reveals diffuse bilateral infi ltrates. On review of her records, type and screen reveal an A+ blood type with a negative antibody screen. Which of the following is the most likely diagnosis? A) Transfusion-related acute lung injury B) Acute hemolytic transfusion reaction C) Febrile nonhemolytic transfusion reaction D) Transfusion-associated circulatory overload E) Transfusion-related sepsis Answer: A This patient has likely developed transfusion-related acute lung injury (TRALI). The patient developed dyspnea, diffuse pulmonary infi ltrates, and hypoxia acutely during the blood transfusion. It usually occurs shortly after the transfusion or can be delayed for several hours. Both the classic and delayed TRALI syndromes are among the most frequent complications following the transfusion of blood products. They are associated with signifi cant morbidity and increased mortality. Antileukocyte antibodies in the donor blood product directed against the recipient leukocytes cause this reaction. TRALI can occur with any blood product. Which of the following is most likely regarding cognitive function in patients such as this? A) Return to baseline in an average of 5 days B) Return to baseline in 2 weeks C) Return to baseline in an average of 30 days D) Return to baseline in an average of 6 months E) Permanent loss of cognitive function Answer: A Postoperative cognitive dysfunction (POCD) is common in adult patients of all ages, recovery in the younger age group is usually within 5 days, and complete recovery is the norm for patients less than 60 years old. Patients older than 60 years of age are at signifi cant risk for long-term cognitive problems, and in this group recovery from POCD may last as long as 6 months and may be permanent. Patients with POCD in all age groups are at an increased risk of all-cause death in the fi rst year after surgery. Most dizziness is benign and is self-limited. Vertigo is often described as an external sensation such as the room is spinning. Vertigo is most commonly from peripheral causes which affect labyrinths of the inner ear. Focal lesions of the brainstem and cerebellum can also lead to vertigo. Vertical nystagmus with a downward fast phase and horizontal nystagmus that changes direction with gaze suggest central vertigo. Signifi cant non-accommodating nystagmus is most often a sign of central vertigo but can occur with peripheral causes as well. In peripheral vertigo, nystagmus typically is provoked by positional maneuvers. It can be inhibited by visual fi xation. Central causes of nystagmus are more likely to be associated with hiccups, diplopia, cranial neuropathies, and dysarthria. 274. You are called to see a 43-year-old female who is 3 days postpartum. She has had a non-complicated pregnancy. She has not been discharged due to feeding issues with her child. She had a normal spontaneous vaginal delivery. This is her fourth vaginal delivery. On physical exam, she has nontender bilateral leg swelling, orthopnea, and a cough with frothy white sputum. Her blood pressure is 150/87 mmHg. Her temperature is 37.2 °C (99.0 °F). She has mild chest pain with inspiration. She has bilateral pulmonary crackles and pitting edema of her lower extremities. WBC is 16,000/μL. CXR is pending. Which of the following is the most likely diagnosis? A) Pulmonary embolism B) Peripartum cardiomyopathy C) Hospital-acquired pneumonia D) Amniotic fl uid embolism E) Acute myocardial infarction 275. A 65-year-old male presents with progressive shortness of breath over the past month. He has a 40-pack-year history of smoking. CT scan of the chest reveals a right middle lobe mass for which he subsequently undergoes biopsy, which reveals adenocarcinoma. Magnetic resonance imaging of the brain reveals a 1-cm tumor in the left cerebral cortex, which is consistent with metastatic disease. The patient has no history of seizures or syncope. The patient is referred to outpatient therapy in the hematology/oncology service as well as follow-up with radiation oncology. The patient is ready for discharge. Which of the following would be the most appropriate therapy for primary seizure prevention? A) Seizure prophylaxis is not indicated. B) Valproate. C) Phenytoin. D) Phenobarbital. E) Oral prednisone 40 mg daily. Answer: A There is no indication for antiepileptic therapy for primary prevention in patients who have brain metastasis who have not undergone resection. Past studies have revealed no difference in seizure rates between placebo and antiepileptic therapy in patients who have brain tumors. Antiepileptic therapy has high rates of adverse reactions and caution should be used in their use. 276. A 78-year-old male is admitted due to swelling over his chest wall. During discussion with the patient, he notes that he had an AICD implanted in the area of the swelling over 3 years ago. His postoperative course had been uneventful and he had never developed any wound dehiscence before. On physical examination, there are palpable swelling and fl uctuance over the right upper chest wall at the site of a well-healed incision. The patient notes some fevers and chills on and off the last few weeks. You are very concerned for a cardiovascular implantable electronic device (CIED) infection. Which of the following is appropriate in the care of your patient? A) Draw two sets of blood cultures before beginning initiation of antimicrobial therapy. B) Percutaneous aspiration of the generator pocket. C) Attempt to preserve the placement of this AICD via empiric antibiotics. D) Request removal of device and obtain gram stain and cultures of the tissue and lead tip. E) A and D. Answer: E A patient with a suspicion of a CIED infection should have two sets of peripheral blood cultures drawn before prompt initiation of antimicrobial therapy. The implantable device should be removed by an expert and the generator-pocket tissue and lead tip should be cultured on explanation. It is appropriate to obtain a transesophageal echocardiogram (TEE) to assess for CIED infection and valvular endocarditis. Percutaneous aspiration is not needed, as the device will be removed. 277. Which of the following occurs in the cognitive function following major cardiac surgery? A) All patients experience some transient cognitive decline. B) Return to baseline can take as long as 6 months. C) Greater declines will be seen in patients with postop delirium. D) Most return to baseline at 5 days. E) All of the above. What is his expected postoperative risk of a major cardiac event? A) 0.5 % B) 1 % C) 5 % D) 10 % E) 20 % Answer: D One of the most widely used preoperative risk assessment tools is the Revised Cardiac Risk Index (RCRI). The RCRI scores patients on a scale from 0 to 6. The patient here has a RCRI score of 3. His score includes high-risk surgery, creatinine greater than 2 mg/dl, and diabetes mellitus requiring insulin. The six factors that comprise the RCRI are high-risk surgical procedures, known ischemic heart disease, congestive heart failure, cerebrovascular disease, diabetes mellitus requiring insulin, and chronic kidney disease with a creatinine greater than 2 mg/dL. 0 predictor = 0.4 %, 1 predictor = 0.9 %, 2 predictors = 6.6 %, ≥3 predictors = >11 % 279. A 24-year-old woman is admitted with signifi cant fatigue, fever, and a sore throat. She reports due to throat pain she has been unable to swallow any liquids for the past 24 h. On physical examination, she is found to have anterior cervical lymphadenopathy, erythematous throat, and mild hepatosplenomegaly. She remembers having mononucleosis in high school. She has mild elevations of her transaminases. Her heterophile antibody test is positive. Which of the following is true concerning the heterophile antibody test? A) Heterophile antibody testing would not be helpful for this patient because the results may be positive owing to her previous episode of mononucleosis. B) She has acute infectious mononucleosis from primary Epstein-Barr virus (EBV On physical exam, she has marked abdominal pain. Her temperature is 39.5 °C (103.0 °F), heart rate is 100 beats per minute, and respirations are 15 per minute. Her blood pressure is 100/62. She has marked hyperactive bowel sounds as well as signifi cant abdominal distention. Laboratory studies include a leukocyte count of 28,000 and hematocrit of 25 %; and blood cultures are negative. Stools are sent for Clostridium toxin which is positive. Which of the following is the most appropriate treatment for the patient's diarrhea? A) Metronidazole orally B) Metronidazole intravenously C) Vancomycin oral D) Vancomycin intravenously Answer: C This patient has severe Clostridium diffi cile -associated diarrhea (CDI). For patients with severe CDI, suitable antibiotic regimens include vancomycin (125 mg four times daily for 10 days; may be increased to 500 mg four times daily) or fi daxomicin (200 mg twice daily for 10 days). Vancomycin has been shown to be superior to metronidazole in severe cases. Fidaxomicin has been shown to be as good as vancomycin, for treating CDI. One study also reported signifi cantly fewer recurrences of infection, a frequent problem with C. diffi cile . Other considerations in this case may be to obtain a CT scan and possible colorectal surgery consultation. 282. A 67-year-old man was admitted with a cerebrovascular accident. He has done well during his hospitalization and is preparing for discharge to a skilled nursing facility. A catheter, which was placed in the emergency room, has been in for 3 days. He reports no prior incident of urinary retention. It is removed, and patient has diffi culty voiding. Which of the following would be considered an abnormal post-void residual (PVR) amount? A) 15 ml B) 50 ml C) 100 ml D) 200 ml E) 300 ml Answer: C Abnormal residual bladder volumes have been defi ned in several ways. No particular defi nition is clinically superior. Some authorities consider volumes greater than 100 mL to be abnormal. Others use a value greater than 20 % of the voided volume to indicate a high residual. In normal adults, the post-void residual volume should be less than 50 ml. Over the age of 60, a range of 50 ml to 100 ml can be seen but is not known to cause signifi cant issues. Post-void residual (PVR) volume increases with age but generally do not rise to above 100 ml unless there is some degree of obstruction or bladder dysfunction. Urinary retention is common after several days of catheter placement, particularly in males. Caution should be used when placing urinary catheters, as they are a signifi cant cause of urinary retention. Whenever possible urinary catheters should be removed. Bladder training and time may improve the retention. Some consideration may be given to starting the male patient on medications to reduce benign prostatic hypertrophy as well. Ultrasound can be used as a noninvasive means of obtaining PVR volume determinations, especially if a precise measurement is not required. The error using this formula, compared with the standard of post-void catheterization, is approximately 21 %. In patients with ascites bedside measurement by ultrasound of PVR can be inaccurate due to an inability to differentiate bladder fl uid from ascitic fl uid. Lisenmeyer TA, Stone JM. Neurogenic bladder and bowel dysfunction. In: De Lisa J, editor. Rehabilitation medicine. Philadelphia: Lippincott-Raven; 1998. p. 1073-106. A 37-year-old male has been admitted for alcohol-related pancreatitis. After six days, he continues with severe midepigastric pain that radiates to the back with nausea and vomiting. He has not been able eat or drink and has not had a bowel movement since being admitted. On physical examination, the temperature is 37.6 °C (99.5 °F), the blood pressure is 120/76 mmHg, the pulse rate is 90 bpm, and the respiratory rate is 20 breaths/ min. There is no scleral icterus or jaundice. The abdomen is distended and with hypoactive bowel sounds. Laboratory studies show leukocyte count 12,400/ μL, amylase 388 μ/L, and lipase 924 μ/L. Repeat CT scan of the abdomen shows a diffusely edematous pancreas with multiple small peripancreatic fl uid collections. Some improvement from the CT scan 3 days ago is noted. He is now afebrile. Which of the following is the most appropriate next step in the management of this patient? A) Enteral nutrition by nasojejunal feeding tube B) Intravenous imipenem C) Pancreatic debridement D) Parenteral nutrition E) Continue with NPO status Answer: A This patient has ongoing moderate pancreatitis. With his possible underlying poor nutritional status due to alcoholism and expected inability to eat, the patient will need nutritional support. This patient will likely be unable to take in oral nutrition for several days.. Enteral nutrition is preferred over parenteral nutrition because of its lower complication rate and proven effi cacy in pancreatitis. Enteral nutrition is provided through a feeding tube ideally placed past the ligament of Treitz so as not to stimulate the pancreas. Broad-spectrum antibiotics such as imipenem therapy are primarily of benefi t in acute pancreatitis when there is evidence of pancreatic necrosis. Randomized, prospective trials have shown no benefi t from antibiotic use in acute pancreatitis of mild to moderate severity without evidence of infection. Pancreatic debridement is undertaken with caution and is not indicated here. Eatock FC, Chong P et al. A randomized study of early nasogastric vs. nasojejunal feeding in severe acute pancreatitis. Am J Gastroenterol. 2005;100:432-9. Eckerwall GE, Axelsson JB, Andersson RG. Early nasogastric feeding in predicted severe acute pancreatitis: a clinical, randomized study. Ann Surg. 2006; 244:959-65. 284 . A 64-year-old female with a past medical history signifi cant for type 2 diabetes mellitus is admitted with increasing shortness of breath. She is admitted for mild congestive heart failure and responds well to therapy. Of note she reports increasing left knee pain. The pain is heightened when she tries to walk with physical therapy. Three months ago she had left knee arthroplasty, and postoperative course was uneventful. Her vital signs are stable. The patient's knee exam reveals a surgical scar but no joint effusion or redness. What should be done next? A) Orthopedics consult B) Arthrocentesis C) Discharged with mild opioid D) Order a knee MRI E) Discharged home with a trial of NSAIDs A 82-year-old female is admitted to the hospital service with urinary tract infection and sepsis. On admission she is noted to be lethargic and unable to swallow medicines. She develops progressive respiratory failure and is intubated. A CXR is consistent with ARDS. An NG tube is placed for administration of medicines. You are considering starting tube feeds in this patient. Which of the following is the most accurate statement regarding enteral tube feeds in this patient? A) Early enteral tube feeds can be expected to reduce her mortality risk. B) The use of omega-3 fatty acids will reduce her mortality risk. C) Enteral tube feeds will increase the risk of infection. D) The benefi ts of early nutrition can be achieved with trophic rates. Answer: D The benefi ts of early enteral tube feedings in the critically ill patient are uncertain. Studies have revealed inconsistent results. There is some suggestion that the incidences of infection can be reduced, but there is no data to suggest long-term mortality improvement. In patients with ARDS, trophic tube feedings at 10 ml/h seem to concur the same benefi t as early full-enteral tube feedings. 286. Which of the following is an acceptable indication for urinary catheter placement? A) A patient who has urinary incontinence and a stage II pressure ulcer B) A patient who is under hospice care and requests a catheter for comfort C) A patient who is delirious and has experienced several falls D) A patient who is admitted for congestive heart failure whose urine output is being closely monitored Answer: B Urinary tract infections (UTIs) are the most common hospitalacquired infections. Most attributed to the use of an indwelling catheter. There should always be a justifi able indication for placement of a urinary catheter, and whenever possible prompt removal should occur. This may be assisted by hospital protocols that trigger automatic reviews of catheter use. 287. An 88-year-old man in hospice care is admitted for dyspnea. He has advanced dementia, severe COPD, and coronary artery disease. He has been in hospice for 2 months. He and his family would like to be discharged to home hospice as soon as possible. He is only on albuterol and ipratropium. On physical examination, he is afebrile, and his blood pressure is 110/76 mmHg, pulse rate is 110 beats/min, and respiratory rate is 28 breaths/min. Oxygen saturation is 90 %. He is cachectic, tachypneic, and disoriented. He is in moderate respiratory distress. Chest examination reveals decreased breath sounds and fi ne inspiratory crackles. In addition to continuing his bronchodilator therapy, which of the following is the most appropriate next step in the treatment of this patient? A) Ceftriaxone and azithromycin B) Morphine C) Methylprednisolone D) Haloperidol E) Lorazepam Answer: B This patient is enrolled in hospice. Every effort should be made to ensure comfort and limit unnecessary treatments. Dyspnea is one of the most common symptoms encountered in palliative care. Opioids are effective in reducing dyspnea in patients with chronic pulmonary disease. A 5-mg dose of oral morphine given four times daily has been shown to help relieve dyspnea in patients with endstage heart failure. Extended-release morphine, starting at a 20 mg given daily has been used to relieve dyspnea in patients with advanced COPD. Bronchodilator therapy should be continued to maintain comfort. Antibiotics and corticosteroids are not indicated. 288. A 59-year-old man presents with fever and a diffuse blistering skin rash. He is recently started on allopurinol for gout. The patient also complains of sore throat and painful watery eyes. On physical examination, the patient is found to have blisters developing over a quarter of his body. Oral mucosal lesions are noted involvement. The estimated body surface area that is currently affected is 15 %. Which of the following statements regarding this patient's diagnosis and treatment are TRUE? A) Immediate treatment with intravenous immunoglobulin has been proven to decrease the extent of the disease and improve mortality. B) Immediate treatment with glucocorticoids will improve mortality. C) The expected mortality rate from this syndrome is about 10 %. D) The most common drug to cause this syndrome is diltiazem. E) Younger individuals have a higher mortality than older individuals with this syndrome. Answer: C This patient has Stevens-Johnson syndrome (SJS). There is no defi nitive evidence that any initial therapy changes outcomes in SJS. Early data suggested that intravenous immunoglobulin (IVIG) was benefi cial, and this traditionally has been the recommended treatment. However, more recent studies have not shown consistent benefi t with IVIG. Immediate cessation of the offending agent or possible agents is necessary. Systemic corticosteroids may be useful for the short-term treatment of SJS, but these drugs increase longterm complications and may have a higher associated mortality. Therapy to prevent secondary infections is important. In principle, the symptomatic treatment of patients with Stevens-Johnson syndrome does not differ from the treatment of patients with extensive burns, and in many instances, these patients are often treated in burn wards. Future studies are required to determine the role of IVIG in the treatment of SJS. The lesions typically begin with blisters developing over target lesions with mucosal involvement. In SJS, the amount of skin detachment is between 10 and 30 % . Mortality is directly related to the amount of skin detachment with a mortality of about 10 % in SJS. Other risk factors for mortality in SJS include older age and intestinal or pulmonary involvement. The most common drugs to cause SJS are sulfonamides, allopurinol, nevirapine, lamotrigine, and aromatic anticonvulsants. 289. A 57-year-old woman with a history of diabetes and familial history of breast cancer is admitted with malaise, an appetite decline, and new-onset ascites. She denies having fevers, chills, diarrhea, nausea, and vomiting. On physical exam, there is no evidence of spider nevi or palmar erythema. Her serum albumin is 3.4 g/ dL. On chest X-ray, a right-sided pleural effusion is noted. A diagnostic paracentesis reveals a glucose of 85 mg/dl, an albumin of 2.8 g/dL, and a WBC of 250/ ul, of which 45 % are neutrophils. Based on the data provided, what is the most likely cause of her ascites? A) Cirrhosis B) Metastatic disease C) Pelvic mass D) Spontaneous bacterial peritonitis E) Tuberculous peritonitis Answer: C Meigs' syndrome is the triad of benign ovarian tumor with ascites and pleural effusion that resolves after resection of the tumor. Typical diagnostic paracentesis reveals a serum-ascites albumin gradient < 1.1 suggesting a nonportal hypertension-mediated process. Of the possibilities for that, ovarian mass is the most likely here. Transdiaphragmatic lymphatic channels are larger in diameter on the right. This results in the pleural effusion being typically classically located on the right side. The etiologies of the ascites and pleural effusion are poorly understood. Further imaging is indicated. Riker D, Goba D. Ovarian mass, pleural effusion, and ascites: revisiting Meigs syndrome. J Bronchology Interv Pulmonol. 2013;20(1):48-51. 290. A 77-year-old female patient presents with dizziness, headache, nausea, and vomiting for the past 48 h. She states that the fl oor feels like it is moving when she walks. The patient is alert, and she tells you she suffered from no recent trauma. On physical exam you note the patient's speech is slightly abnormal. During the neurological examination, the patient is able to understand your questions, respond appropriately, and repeat words, but her words are poorly articulated. She has a great deal of diffi culty walking across the room without assistance. What is your next step in the management of this patient? A) Administer unfractionated heparin B) Epley maneuver C) CT scan without contrast D) Emergent MRI or MRA E) Observation alone Answer: D This patient has central vertigo possibly due to a cerebellar infarction. Multiple cerebellar signs are noted which help distinguish this from benign peripheral vertigo. Due to obstruction by a posterior fossa bone artifact, CT scan may not be of benefi t. Emergent MRI and MRA if available are the tests of choice. This should be done to confi rm the diagnosis and followed for the development of an obstructing hydrocephalus, which can occur with cerebellar infarction. Since the posterior fossa is a relatively small and nonexpandable space, hemorrhage or edema can lead to rapid compression. Early neurosurgical consultation should be considered. On physical examination, temperature is normal. Blood pressure is 147/83, pulse rate is 70 beats/min, and respiratory rate is 12 breaths/min. Other physical examination fi ndings are within normal limits. Which of the following is the most appropriate insulin therapy after surgery? A) Continuous intravenous insulin infusion B) Previous schedule of 70/30 insulin C) Subcutaneous insulin infusion D) Insulin glargine once daily and insulin aspart before each meal E) Sliding-scale insulin alone F) Insulin aspart before each meal alone Answer: D This patient should receive basal insulin as well as scheduled insulin before each meal. This should be adjusted for conditions that occur in the hospital. A patient with longstanding type 1 diabetes makes no endogenous insulin and requires a maintenance dose of insulin postoperatively. It is expected that her PO intake would be markedly decreased, and subsequently her insulin dose should be decreased. 293. You are urgently called to see in consultation of a 36-year-old woman who is in postop recovery. She has a sudden elevation of her temperature and is thought to be septic. Her laparoscopic cholecystectomy was completed 45 min ago without complication. On physical exam her temperature is 40.5 °C (105 °F). She has respiratory rate of 28 breaths per minute. She is tachycardic, shaking, and confused. There is diffuse muscular rigidity noted. Which of the following drugs should be administered immediately? A) Acetaminophen B) Haloperidol C) Hydrocortisone D) Ibuprofen E) Dantrolene Answer: E The patient has malignant hyperthermia. Dantrolene should be given. Physical cooling in addition to dantrolene with cooling blanket or IV fl uids should be used as well. Dantrolene may be used in other central causes of extreme hyperthermic such as neuroleptic malignant syndrome. In this case, the episode was probably caused by succinylcholine and/or inhalational anesthetic. This syndrome occurs in individuals with inherited abnormality of skeletal muscle sarcoplasmic reticulum. More than 30 mutations account for human malignant hyperthermia. Genetic testing is available to establish a diagnosis. The caffeine halothane contracture test remains the criterion standard. This is a muscle biopsy and performed at a designated center. The syndrome presents with hyperthermia or a rapid increase in body temperature that exceeds the ability of the body to lose heat. Muscular rigidity, acidosis, cardiovascular instability, and rhabdomyolysis also occur. Antipyretics such as acetaminophen, ibuprofen, and corticosteroids are of little use. The dantrolene dose is 2.5 mg/kg rapid IV bolus and may be repeated PRN. Occasionally a dose up to 30 mg/kg is necessary. Which of the following is the most appropriate perioperative recommendation regarding anticoagulation in this patient? A) Discontinue warfarin 5 days before surgery and bridge with full-dose IV heparin before and after surgery. B) Discontinue warfarin 5 days before surgery and restart on the evening of the surgery. C) Continue with warfarin. D) Reverse anticoagulation with fresh frozen plasma transfusion 1 h before surgery and restart warfarin on the evening of the surgery. In patients with mechanical valves and at low risk for thromboembolism, low-dose low-molecular-weight heparin or no bridging is recommended. The short-term risk of anticoagulant discontinuation in this patient is small. The current recommendation is to stop warfarin 5 days before the procedure. The INR goal is 1.5. Warfarin should be restarted within 24 h after the procedure. In patients with a mechanical valve and an increased risk of a thromboembolic event, it is recommended that unfractionated heparin be begun intravenously when the INR falls below 2.0. This should be stopped 4-5 h before the procedure and restarted after surgery. In patients with a mechanical heart valve who require emergent surgery, reversal with fresh frozen plasma may be performed. 297. An 88-year-old female who was admitted to the hip fracture service for a right hip fracture has currently become agitated and confused. She underwent hip fracture repair two days prior. She has a history of osteoporosis, dementia, and type 2 diabetes. Her postoperative medicines include oxycodone 5 mg every 4 h as needed for pain as well as IV morphine 1-2 mg/h as needed for the pain. During the patient's fi rst night, she was calm and relatively free of pain. However, on her second night, she has become acutely agitated and is reported by the nurse to be screaming and pulling out lines and drains. Her temperature is 99.1 °F. Her pulse rate is 100 beats/min. Her respirations are 20 per minute. Her oxygenation is 92 % on room air. Her hematocrit and hemoglobin are within normal limits as well as the rest of her electrolytes. Which of the following is the appropriate response/ treatment for this patient's delirium? A) Four-point restraints B) One 2 mg dose of intravenous lorazepam C) One 5 mg dose of oral haloperidol D) One 0.5 mg dose of oral haloperidol E) One 5 mg dose of intravenous haloperidol Answer: D Treatment of postoperative-induced delirium is a common issue confronted in the hospital setting. Delirium that causes injury to the patient or others should be treated with medications. This can be a diffi cult management issue. No medication is currently approved by the Food and Drug Administration for the treatment of delirium. Current guidelines recommend using low-dose antipsychotics such as haloperidol. The use of benzodiazepines should be limited, unless concurrent alcohol withdrawal is present. A specifi c FDA warning has been issued for intravenous haloperidol due to the risk of torsades de pointes in 2007. Low-dose haloperidol, less than 2 mg, has a low incidence of extrapyramidal side effects. QTc prolongation monitoring is recommended for patients. If feasible, this patient should have had a baseline EKG as well as a follow-up EKG. Haloperidol at doses greater than 4.5 mg increases the incidence of extrapyramidal side effects and should be avoided. The surgery was uneventful. On hospital day 2, she has a sudden onset of tachypnea and hypoxemia. A computed tomography pulmonary angiogram reveals a thrombus in the pulmonary artery to the right lower lobe. Her INR is 1.0. What is the most likely cause of her thrombosis? A) Surgery-induced thrombosis B) Depletion of thrombin due to the surgical acutephase response C) Thrombogenesis due to postoperative hypovolemia D) Undetected prior thrombus E) Rebound hypercoagulability and subsequent thromboembolism Answer: E Rebound hypercoagulability is the most likely cause. This may occur after abrupt cessation of warfarin. In addition, surgery increases the risk of thromboembolic events. Following an abrupt withdrawal of warfarin, thrombin and fi brin formation increase and very high levels of thrombin activation are seen. If possible, warfarin withdrawal should be gradual which would not have been feasible in the current case. Safely resuming anticoagulation after surgery should be a goal as well. A 60-year-old man who has metastatic lung cancer and painful bone metastases reports severe pruritus that started when he began to take morphine for his pain. Pain in his chest wall and legs has been successfully treated with sustained-release morphine (80 mg every 12 h) and short-acting morphine (15 mg orally every 2 h as needed for breakthrough pain) which he uses two or three times daily, depending on his level of activity. On physical examination, the temperature is 37 °C (98.6 °F), pulse rate is 80 beats per minute, respirations are 16 per minute, and blood pressure is 115/70 mmHg. Oxygen saturation by pulse oximetry is 95 % on room air. The patient is alert and oriented. His pupils are 4 mm initially and constrict to 2 mm with a light stimulus. The lungs are clear. Cardiac examination shows a normal rate and regular rhythm. No rash is seen. Examination of the abdomen is signifi cant for suprapubic dullness and sensitivity. Neurological examination is nonfocal. Which of the following should be done next? A) Change to oxycodone, 40 mg every 12 h, and oxycodone, 5-10 mg every 2 h as needed B) Lower the dosage of sustained-release morphine to 30 mg every 12 h C) Continue with same morphine dose D) Change to oxycodone, 60 mg every 12 h, and oxycodone, 15 mg every 2 h as needed Answer: A Oxycodone may cause somewhat less nausea, hallucinations, and pruritus than morphine. Mild to moderate morphineinduced puritis may be managed by small-dose reductions or antihistamines. This patient has severe puritis which may be relieved by changing to oxycodone. The patient's baseline long-acting morphine daily dose was 160 mg, with a minimum short-acting morphine dose of 30 mg daily, which yields a total daily dose of 190 mg. The morphine-to-oxycodone ratio is 1.5:1. This patient's morphine-equivalent daily dose of oxycodone would be 120 mg. The daily dose of oxycodone would be 60 mg. Thus, the every-12-h dose of long-acting oxycodone would be 40 mg. A 68-year-old female who has metastatic small cell lung cancer presents to the emergency room with shortness of breath. She is noted to be in marked respiratory distress and is intubated by emergency room personnel. She is admitted to the intensive care unit. On review of the medical records, you fi nd that the patient has an advanced directive, which indicates that the patient did not want to be intubated. This is noted both in a signed advanced directive as well as in the hospital records. You arrange a family meeting to discuss goals of care. The patient's daughter has recently quit her job and has moved in with her mother to provide care. You discuss the case with her, and she states that her mother has changed her mind recently and would like to be on the ventilator at all costs. Which of the following is the correct course of action? A) Follow the patient's written documentations and extubate the patient and provide comfort care. B) Follow the daughter's instructions and have patient remain intubated. C) Request an ethics consultation. D) Consult the hospital's legal affairs department. Answer: C It is of primary importance to follow the patient's wishes. In this particular case, there is some diffi culty in determining if the patient has recently changed her mind, as is suggested by the daughter. She has clearly documented her advance directives, and it would be appropriate to withdraw life support if the daughter did not provide the confl icting statement. Financial confl icts of interest often interfere with the surrogates ability to act in the best interest of the patient. In this particular case, there are circumstances that suggest that fi nancial considerations may be infl uencing the statement. It would be diffi cult for an individual practitioner to make this determination, without the potential of liability. Subsequently, an ethics consultation would be the correct course of action. As there are several factors, ethics and clinical, involved, an attorney alone would not be in a position to resolve the issue. 303. An 83-year-old female is admitted from a nursing home to the hospital for shortness of breath. On chest X-ray, she has a new-onset pleural effusion for which thoracentesis is indicated. On her medical record, it is reported that she has a history of dementia. On physical exam she is awake and alert. She knows that she is in the hospital, knows her name and address, but is confused about the current date. On review of her medical records, you discover that she has neither family members nor a durable power of attorney. In attempting to obtain consent for the procedure, which of the following is the next best step? A) Proceed without consent. B) Assign guardianship. C) Determine capacity yourself. D) Psychiatric consultation for competency. E) Ethics consultation. Answer: C There are four components of determining capacity in decision-making concerning a particular treatment or test: (1) an understanding of relevant information about proposed diagnostic tests or treatment, (2) appreciation of their medical situation, (3) using reason to make decisions, (and 4) ability to communicate their choice. In most instances, the primary physician should possess the ability to determine capacity. Capacity is not the same measurement as competence. Competence is determined by a court of law and uses issues of capacity in evaluating the legal ability to contract. A psychiatric consultation can determine competency but is usually not needed to determine capacity. Assigning guardianship or an ethics consultation can be a lengthy process and should be reserved for cases with signifi cant issues to be resolve. A 65-year-old male is admitted to the hospital for elective total knee arthroplasty. He has a history of type 2 diabetes mellitus and is treated with metformin. He reports fair glucose control with diet and oral agents. He has never been on insulin. On physical examination he has mild edema of his lower extremities but otherwise is within normal range. Preoperative laboratory studies have been done 1 week prior. His hemoglobin A1c revealed a concentration of 6.8 %. Plasma glucose level measured on the day of surgery is 210 mg/L. Which of the following is the most appropriate treatment for patients with elevated blood sugars preoperatively and postoperatively? A) Metformin B) Sliding-scale insulin C) IV hydration D) Basal and sliding-scale insulin E) Diet control alone Answer: D The goal of glycemic control in the hospitalized patient is balancing the risks of hypoglycemia against the known benefi ts in morbidity and mortality. Although tight control has been advocated in the past, current consensus guidelines recommend less stringent glycemic goals, typically between 80 and 150 mg/dL. The ultimate goal in the management of diabetic patients (DM) is to achieve outcomes equivalent to those in patients without DM. A meta-analysis of 15 studies reports that hyperglycemia increased both in-hospital mortality and incidence of heart failure in patients admitted for acute myocardial infarction. Several other studies have also demonstrated the benefi ts of glycemic control in the perioperative area. Type 2 diabetes mellitus often requires insulin while in the hospital. The requirements may be unpredictable. This may be due to the stress of hospitalization, dietary changes, glucose added to IV fl uids, and medicine interactions. Sliding scale alone has often been traditionally used in the past. However, this method of control often results in wide fl uctuations in glycemic control. The optimal plasma glucose level postoperatively is not known, and certainly tight control has its risks. A 55-year-old female has been admitted for cellulitis. She has responded well to antibiotics and is ready for discharge. On admission she was noted to be in atrial fi brillation. She has been treated with low-molecularweight heparin in the hospital. She fi rst noted the irregular heartbeat 4 weeks ago. She denies chest pain, shortness of breath, nausea, or gastrointestinal symptoms. Past medical history is unremarkable. There is no history of hypertension, diabetes, or tobacco use. Her medications include metoprolol. On physical examination, she has a blood pressure of 124/76 mmHg and a pulse of 70 beats/min. An echocardiogram shows a left atrial size of 3.5 cm. Left ventricular ejection fraction is 63 %. There are no valvular or structural abnormalities. Which of the following would be the appropriate treatment of her atrial fi brillation? A) She requires no antiplatelet therapy or anticoagulation because the risk of embolism is low. B) Lifetime warfarin therapy is indicated for atrial fi brillation in this situation to reduce the risk of stroke. C) She should be started on IV heparin and undergo electrical cardioversion. D) She should continue on SC low-molecular-weight heparin and transitioned to warfarin. E) Her risk of an embolic stroke is less than 1 %, and she should take a daily aspirin. Answer: E Patients younger than 60 years of age without structural heart disease or without risk factors have a very low annual risk of cardioembolism of less than 0.5 %. Therefore, it is recommended that these patients only take aspirin daily for stroke prevention. The risk of stroke can be estimated by calculating the CHADS2 score. Older patients with numerous risk factors may have annual stroke risks of 10-15 % and must take a vitamin K antagonist or alternate indefi nitely. Cardioversion may be indicated for symptomatic patients who want an initial opportunity to remain in sinus rhythm. A) PEG tubes reduce aspiration as opposed to nasogastric tubes. B) In end-stage advanced malignancy with cachexia, PEG tubes have been proven to improve survival and reduce morbidity. C) PEG tubes have been proven to improve survival in end-stage dementia. D) Mean survival after PEG tube placement for failure to thrive is 6 months. Answer: D The physician is often faced with this decision in a variety of end-of-life situations to consider placement of a PEG tube. Survival benefi ts of PEG tube placement are often minimal at best. There is a wide range of cultural expectations in reference to this issue. It is important to understand the facts concerning the possible benefi ts or lack of benefi ts of PEG tube placement when counseling the patient and family. As noted in this question, survival benefi ts for PEG tube placement in a patient with failure to thrive to variety of conditions are modest at best. A 59-year-old man has been admitted for congestive heart failure. His symptoms have resolved. Prior to discharge the cardiology service would like him to undergo placement of an automatic implantable cardiac converter defi brillator (AICD). He is on warfarin with an INR of 2.9. His other problems include rate-controlled atrial fi brillation and coronary artery disease. An echocardiogram performed 2 weeks ago showed a left ventricular ejection fraction of 25 % and a well-functioning mechanical mitral valve. Trace edema is noted in the extremities. How should his warfarin be managed prior to placement of his AICD? A) Continue warfarin, with a target INR of 3.5 or less on the day of the procedure. B) Discontinue warfarin 5 days before the procedure and resume the day after the procedure. C) Discontinue warfarin 5 days before the procedure and bridge with an unfractionated heparin infusion. D) Discontinue warfarin 5 days before the procedure and bridge with low-molecular-weight heparin. Answer: A Not all procedures require warfarin to be stopped. In some cases, there is data to support continuing warfarin as opposed to bridging therapy. A randomized, controlled trial found that patients at high risk for thromboembolic events on warfarin who need a pacemaker or implantable cardioverter defi brillator (ICD) can safely continue warfarin without bridging anticoagulation. Continuing warfarin treatment at the time of pacemaker in patients with high thrombotic risk was associated with a lower incidence of clinically signifi cant device-pocket hematoma, as opposed to bridging with heparin. A 56-year-old male is admitted to the hospital with fever and cough. He was well until 1 week before admission when he noted progressive shortness of breath, cough, and productive sputum. On the day of admission, the patient's wife noted him to be lethargic. The past medical history is notable for alcohol abuse and hypertension. On examination, the patient is lethargic. Temperature is 38.9 °C (102 °F), blood pressure is 110/85 mmHg, and oxygen saturation is 86 % on room air. There are decreased breath sounds at the right lung base. Heart sounds are normal. The abdomen is soft. There is no peripheral edema. Chest radiography shows a right lower lobe infi ltrate with a moderate pleural effusion. The white blood cell count is 15,000/μL and 6 % bands. He is admitted and started on broad-spectrum antibiotics. On hospital day 3 he is not eating due to lethargy. A nasogastric tube is inserted, and tube feedings are started. The next day, plasma phosphate is found to be 1.2 mg/dL and calcium is 9.2 mg/dL. What is the most appropriate approach to correcting the hypophosphatemia? A) Administer IV calcium gluconate 1 g followed by infusion of IV phosphate at a rate of 8 mmol/h for 6 h. B) Administer IV phosphate alone at a rate of 4 mmol/h for 6 h. C) Administer IV phosphate alone at a rate of 8 mmol/h for 6 h. D) Stop tube feedings, phosphate is expected to normalize over the course of the next 24-48 h. E) Initiate oral phosphate replacement at a dose of 1750 mg/day. Answer: C Severe hypophosphatemia occurs when the serum concentration falls below 2 mg/dL . In this circumstance, IV replacement is recommended. In this patient with a level of 1.2 mg/dL, the recommended infusion rate is 8 mmol/h over 6 h for a total dose of 48 mmol. Levels should be checked every 6 h as well. Malnutrition from fasting or starvation may result in depletion of phosphate. When nutrition is initiated, redistribution of phosphate into cells occurs. This is common in alcoholics. It is generally recommended to use oral phosphate repletion when the serum phosphate levels are greater than 1.5-2.5 mg/dL. A 58-year-old male is admitted to the hospital for elective hip replacement therapy. He has a history of chronic pulmonary disease and takes inhaled steroids as well as albuterol inhalers. He was admitted to the hospital 2 weeks ago for a moderate exacerbation of COPD for which he recently completed a 10-day course of prednisone. He is currently asymptomatic, and his breathing is back to baseline. He states that he has not taken steroids within the past year other than his recent admission. You are asked to provide clearance for the orthopedic service of this patient. Which of the following is the most appropriate treatment? A) Obtain a Cortrosyn stimulation test and begin steroids if there is evidence of cortisol defi ciency. B) Administer intravenous hydrocortisone 50 mg on the morning of surgery. C) Administer intravenous hydrocortisone 100 mg preoperatively and then 50 mg every 8 h for 2 days after surgery. D) Proceed with surgery. E) Postpone surgery for 2 weeks. 316. An 86-year-old male is admitted for cough, dyspnea, and dysphagia. He has a known large non-small cell cancer in the upper lobe of the right lung and is on week 4 of palliative irradiation. He reports anorexia, diffi culty swallowing solid food, and right shoulder pain. His wife and family are concerned about dehydration. They request IV fl uids and nutrition. On physical examination, the patient is thin and appears weak but alert. Pulse rate is 120 beats per minute, respirations are 24 per minute, and blood pressure is 150/70 mmHg. There are temporal wasting and a dry oropharynx. The patient's breathing is shallow, with mild tachypnea. Breath sounds are diminished in the upper lobe of the right lung. You convene a family meeting to discuss options. Which of the following would be the most likely outcome of intravenous hydration or nutrition in this patient? A) Reduced BUN/serum creatinine ratio B) Prolonged survival C) Increased albumin level D) Improved quality of life Answer: A Families feel an important obligation to provide nutrition and hydration to the dying patient. A randomized controlled trial found that parenteral hydration did not improve quality of life in advanced cancer. The intravenous fl uids would likely reduce this patient's prerenal azotemic state in the short term but would not have a benefi cial impact on his quality of life. These facts can guide counseling of patients and families in seeking noninvasive measures for this stage of advanced cancer. 318. A 26-year-old woman is evaluated in the emergency department for abdominal pain. She reports a vague loss of appetite for the past day and has had progressively severe abdominal pain at her umbilicus. The pain is collicky. She reports that she is otherwise healthy and has had no sick contacts. Surgery has been consulted and recommends observation. You are consulted for admission. On physical exam her temperature is 38.2 °C (100.8 °F), heart rate 110 bpm, and otherwise normal vital signs. Her abdomen is tender in the right lower quadrant and pelvic examination performed in the emergency room is normal. Urine pregnancy test is negative. Which of the following imaging modalities would you do next? A) Colonoscopy B) Pelvic ultrasound C) CT of the abdomen without contrast D) Ultrasound of the abdomen E) Transvaginal ultrasound F) Plain fi lm of the abdomen Answer: C CT scan is indicated for the diagnoses of acute appendicitis. It has been shown to be superior to ultrasound or plain radiograph in the diagnosis of acute appendicitis, The appendix is not always visualized on CT, but nonvisualization of the appendix on CT scan is associated with surgical fi ndings of a normal appendix 98 % of the time. This patient presented with classic fi ndings for acute appendicitis. Initial anorexia progressed to vague periumbilical pain. This was followed by localization to the right lower quadrant. Low-grade fever and leukocytosis may be present. Acute appendicitis is primarily a clinical diagnosis. However, imaging modalities are frequently employed as the symptoms are not always classic and take time to evolve. Plain radiographs are rarely helpful. Ultrasound may demonstrate an enlarged appendix with a thick wall, but is most useful to rule out gynecological disease such as ovarian pathology, tuboovarian abscess, or ectopic pregnancy, which can mimic appendicitis. An abdominal and pelvic computed tomography scan shows a large amount of stool but no bowel obstruction. Which of the following is the correct treatment for this patient's ongoing constipation? A) Add lactulose. B) Add N-methylnaltrexone. C) Add docusate. D) Place a nasogastric tube for bowel decompression. E) Request a colorectal surgery consult for manual disimpaction. Answer: A Constipation is the most frequent side effect associated with long-term opioid therapy. Osmotic laxatives, such as mannitol, lactulose, and sorbitol, are effective in the palliation of opioid-induced constipation. Although expert consensus supports the use of prophylactic bowel regimens in all patients taking opioids, little evidence demonstrates the effi cacy of one regimen over another. Bulk-forming laxatives increase stool volume but should be used with caution in patients with advanced cancer because they require adequate fl uid intake and physical activity to prevent exacerbation of constipation. Docusate has very little effect when given alone for opioidinduced constipation. Gastric motility is decreased in these patients and softening of the stool alone may not alleviate the symptom. In many situations, its effi cacy has been questioned. N-methylnaltrexone is used for the treatment of opioidinduced constipation in patients with advanced illness who are receiving palliative care, when response to laxative therapy has been insuffi cient. In this patient adding, starting and continuing with lactulose is the next step. In addition a bowel diary may be beneficial to review on her follow-up appointment. Pappagallo M. Incidence, prevalence, and management of opioid bowel dysfunction. Am J Surg. 2001;182 (suppl 5A):11S-8S. A 53-year-old woman who has hepatitis C cirrhosis is admitted for worsening ascites. In addition to complaints of abdominal pain, she complains of severe puritis. She has been on cholestyramine for several months for the itching. On physical exam multiple excoriations of her skin are noted and she is unable to stop scratching. She is very anxious and fatigued. Her serum laboratory results are stable from last admission, including a stable total bilirubin. Ultrasonography shows no evidence of biliary ductal dilatation or changes in her liver. Which of the following should you now recommend? A) Ursodeoxycholic acid at 30 mg/kg daily B) Diphenhydramine 50 mg every 6 h C) Naltrexone 25 mg daily D) Morphine 5 mg BID E) Hydroxyzine 10 mg BID Answer: C Refractory itching is a common in end-stage liver disease patients. It may be severe leading to signifi cant excoriations. Cholestyramine has been the mainstay of treatment. Patients who do not respond to continued doses of cholestyramine probably will not respond to an antihistamine. Naltrexone is tolerated well and is a reasonable option in these cases. Patients started on naltrexone should be followed for signs of withdrawal. Wolfhaqen FH, Sternieri E, Hop WC et al. Oral naltrexone therapy for cholestatic pruritus: a double-blind, placebocontrolled study. Gastroenterology. 1997; 113:1264-9. 322 . A 69-year-old female with osteoarthritis of the knees for many years and has been advised by her orthopedist that the timing is now right to undergo knee arthroplasty. She has a history of diabetes, high cholesterol, hypertension, and coronary artery disease. Nine months ago, she underwent a drug-eluting stent placement for worsening angina, which she tolerated well. She has been angina-free since that time and is able to walk up several fl ights of stairs without angina. Current medications are aspirin, clopidogrel, losartan, and metoprolol. Your recommendations concerning surgery are the following: A) Surgery can proceed as planned. B) Surgery should wait for 2 months. C) Surgery can occur in 3 months. D) Surgery can occur in 9 months. Answer: C Elective surgery should be delayed at least 1 year after the placement of a drug-eluting stent. Rapid thrombosis of a drug-eluting stent (DES) is a catastrophic complication. The risk of stent thrombosis is increased in the perioperative setting and is strongly associated with the cessation of antiplatelet therapy. To avoid thrombosis with DES, aspirin and antiplatelet agents should be continued throughout surgery. In spite of the increased risk of bleeding, this strategy is acceptable in many types of invasive surgical procedures with no change in outcome. In situations where surgery may be needed on a semi-urgent basis in patients who have received a drug-eluding stent within 1 year and the risk of bleeding is high. In these situations, consultation with cardiology is recommended. Elective surgery with bare metal stents should be delayed for 30-90 days. A patient with severe dementia is admitted for worsening anorexia and nausea over the past 6 weeks. She lives at home with her family. The family would like to continue palliative care but are looking to improve her appetite and diminish her nausea. You and the family meet and agree on a conservative course of action. Which of the following statements accurately characterizes the treatment of these complications of severe dementia? A) Haloperidol has minimal effects against nausea. B) Even though this patient has severe dementia, it would be unethical to withhold nutrition and hydration. C) A feeding tube will reduce the risk of aspiration pneumonia. D) A trial of antidepressants is indicated. E) Impaction may explain all the symptoms. F) A trial of megestrol acetate. Answer: E Anorexia and gastrointestinal symptoms are common near the end of life. Despite a nonaggressive approach, some simple measures may improve symptoms. Haloperidol may be highly effective against nausea and may be less sedating than many commonly used agents, such as prochlorperazine. Impactions are common and can present with a variety of symptoms. Treatment can be relatively easy and can improve comfort. Because of the terminal and irreversible nature of end-stage dementia and the substantial burden that continued lifeprolonging care may pose, initiating aggressive hydration and nutrition would not be indicated. Appetite stimulants such as megestrol acetate have not been shown to be of any benefi t in the anorexia of end-stage dementia. Hanson LC, Ersek M, Gilliam R, Carey TS. Oral feeding options for patients with dementia: a systematic review. J Am Geriatr Soc. 2011;59(3):463-72. A 23-year-old female is admitted with a new deep venous thrombosis (DVT). She is pregnant and in her late second trimester. You are consulted for management of her DVT. In review of her labs, it is noticed that her liver functions are elevated. Her AST is 120 units/L; her ALT is 140 units/L. T. bili is 1.6 mg/dL. Which of the following is the likely diagnosis? A) Hyperemesis gravidarum B) HELLP C) Cholestasis of pregnancy D) Acute fatty liver of pregnancy E) None of the above Answer: C Gestational age of the pregnancy is a great guide to the differential of liver disease in the pregnant woman. Cholestasis of pregnancy is common and most typically presents in the late second trimester. Approximately 1 % of pregnancies in the United States are affected by this condition. Some hepatic diseases of pregnancy are mild, and some require urgent and defi nitive treatment. A common condition of the fi rst trimester is hyperemesis gravidarum and may result in elevated AST and ALT; however this usually resolves by week 20 of gestation. Acute fatty liver of pregnancy is a cause of acute liver failure that can develop in the late second or third trimester. Elevated LFTs and bilirubin are most commonly seen. Although symptoms and signs are similar to those of preeclampsia and HELLP syndrome, aminotransferase levels tend to be much higher. Riely CA. Liver disease in the pregnant patient. Am J Gastroenterol. 1999; 94:1728-32. 326 . A 66-year-old male is admitted with acute onset of left hemiplegia. He has a history of hypertension, nonvalvular atrial fi brillation, and thyroid disease. He has been lost to medical follow-up in recent years and has been on no anticoagulation. On physical exam, motor strength is 1/5 in the left arm and 2/5 in the left leg. Electrocardiogram reveals atrial fi brillation with a heart rate of 70 beats per minute. MRI performed on presentation reveals a right middle cerebral artery infarction. Which of the following is appropriate treatment for stroke prevention? A) Aspirin 350 mg daily alone B) Clopidogrel 25 mg daily C) Warfarin, adjusted to achieve an INR of 2-3 D) Unfractionated heparin bolus, followed by infusion E) Enoxaparin Answer: C Guidelines do not support the routine use of anticoagulation for acute ischemic stroke. In this particular case with a large territory middle cerebral artery infarct, any urgent anticoagulation may increase the risk of conversion to hemorrhage. Several randomized, controlled trials that used heparin early after ischemic stroke failed to show a signifi cant overall benefi t of treatment over controls. An exception may be in patients with acute ischemic stroke ipsilateral to a severe stenosis or occlusion of the internal carotid artery. Stroke prevention treatment for atrial fi brillation is most often determined according to the CHADS2/CHADS2VAS system. Warfarin continues to be the most commonly used agent, although a number of newer agents including dabigatran are increasingly being prescribed. Current recommendation is that warfarin be started during the hospitalization. Bridging with low-molecular-weight heparin is not usually needed but may be considered in certain circumstances. A 37-year-old male with a history of intravenous drug abuse is admitted with fever and hypertension. A diagnosis of mitral valve endocarditis is made by echocardiogram. He is noted to have a large lesion on his mitral valve with moderate regurgitation. He is started on broadspectrum antibiotics and has a clinically good response. When is surgery indicated in the presence of endocarditis? A) Heart failure B) After several embolic events C) Myocardial abscess D) Confi rmed fungal endocarditis E) All of the above Answer: E Fifteen to twenty percent of the patients who have endocarditis will ultimately require surgical intervention. Congestive heart failure in a patient with native valve endocarditis is the primary indication for surgery. The decision to proceed with surgery is often diffi cult due to patient comorbidities. Traditional criteria include those listed above. It is suggested that surgery may be considered in patients with large lesions and signifi cant valvular disease. Early surgery reduces the risk of embolic events, although this has not been proven to change overall mortality. Failure of medical treatment is another indication for surgery, although guidelines are not specifi c. In addition surgery should be considered in patients with multiresistant organisms. Endocarditis in many circumstances warrants early cardiothoracic surgery consultation. 328. Which of the following patients with metastatic disease is potentially curable by surgical resection? A) A 22-year-old man with a history of osteosarcoma of the left femur with a 1-cm metastasis to his right lower lobe B) A 63-year-old woman with a history of colon cancer with one metastases to the left lobe of the liver C) Operable non-small cell lung cancer with a single brain metastasis D) All of the above E) None of the above Answer: D In colon, non-small cell lung and osteosarcoma cancer cures have been reported with resection of solitary metastatic lesions. Metastases typically represent widespread systemic dissemination of disease and are associated with poor prognosis. Palliative chemotherapy is generally the accepted method of treatment. Over the last several years, numerous reports and studies have demonstrated long-term survival after resection of isolated metastasis. After extensive investigation for further metastatic sites, isolated metastasis should be considered for reaction in select cases. Manfredi S, Bouvier AM, Lepage C et al. Incidence and patterns of recurrence after resection for cure of colonic cancer in a well defi ned population. Br J Surg. 2006; 93:1115-22. 329 . A 56-year-old white male with known clinical atherosclerotic disease is admitted with severe leg cramps. His past medical history is signifi cant for a myocardial infarction (MI) 4 years ago requiring stent placement. At the time of his MI, he was initiated on a high-intensity statin; since then he has developed severe leg cramps. What would be the next best alternative in lipid therapy for this patient? A) Start atorvastatin 20 mg PO daily. B) No longer a need for statin therapy since his MI was 4 years ago. C) Start rosuvastatin 20 mg PO QHS. D) Start pravastatin 10 mg PO QHS. Answer: A He should be on a high-intensity statin, but he was unable to tolerate the side effects. According to American College of Cardiology guidelines, patients with known clinical atherosclerotic disease should be on a moderate-intensity statin if not a candidate or cannot tolerate the highintensity regimen. Atorvastatin 20 mg is a moderateintensity statin. The moderate-intensity daily dose will lower LDL-C by approximately 30 to <50 %, whereas the high-intensity therapy lowers LDL-C by approximately ≥50 %. Lastly, pravastatin 10 mg is a low-intensity statin. 330. A 62-year-old man is admitted for dehydration. He also reports severe nausea and vomiting that began 24 h ago. He recently started chemotherapy for non-small cell lung cancer. His last dose was 48 h ago. On physical examination his abdomen is soft and nontender. Bowel sounds are present. He is admitted and started in intravenous fl uids. Despite several doses of ondansetron, he continues to have near constant nausea. What would be the next appropriate treatment for his nausea and vomiting? A) Dexamethasone B) Haloperidol C) Lorazepam D) Octreotide Answer: A Dexamethasone is recommended for the management of delayed chemotherapy-induced nausea and vomiting. Delayed nausea and vomiting are any nausea and vomiting that occurred after the day that chemotherapy is infused. Nausea and vomiting are two of the most feared cancer treatment-related side effects for cancer patients. Dexamethasone has synergistic action with many antiemetic medications. Its specifi c antiemetic mechanism of action is not fully understood. It is generally started at 8 mg once or twice daily. Corticosteroids may be effective as monotherapy as well. A 63-year-old man is admitted to the hospital because of hematemesis. He has gastroesophageal refl ux disease and atrial fi brillation; he takes warfarin. He had felt well until this morning when nausea developed after eating. He vomited blood once and was brought to the hospital. On physical exam, the temperature is normal. Pulse rate is 84 beats per minute and irregular, and blood pressure is 112/74 mmHg. Abdominal examination is normal. Hemoglobin is 11.8 g/dL, serum creatinine is 0.9 mg/dL, and eGFR is greater than 60 mL/ min/1.73 m 2 . Intravenous isotonic saline is given, and nasogastric lavage is subsequently performed. Upper endoscopy reveals a duodenal ulcer, which is successfully cauterized. Warfarin is discontinued, and intravenous pantoprazole is begun. No additional bleeding is noted after 48 h, and the patient is prepared for discharge. How long after the bleeding episode can this patient's warfarin be safely restarted? A) One week. B) One month. C) Six weeks. D) Three months. E) Warfarin should not be restarted. Answer: A Gastrointestinal (GI) bleeding affects an estimated 4.5 % of warfarin-treated patients annually and is associated with a signifi cant risk of death. These patients present a dilemma for clinicians regarding when to restart warfarin. A recent study examined patients who had GI bleeds when on warfarin. They found that warfarin therapy resumption within 1 week after a GI bleed was, after 90 days, associated with a lower adjusted risk for thrombosis and death without signifi cantly increasing the risk for recurrent GI bleeding compared to those who did not resume warfarin. The median time to restart warfarin was 4 days. From this study, a reasonable period of 7 days is suggested. 332. An 82-year-old male is admitted for communityacquired pneumonia. During the fi rst 24 h of admission, he undergoes cardiopulmonary arrest. He was subsequently successfully coded on the fl oor. The family cannot be contacted, and full resuscitation measures are taken. He is transferred to the ICU. Which of the following will characterize the patient's post-arrest clinical course? A) Increased intracranial pressure B) Intact cerebrovascular autoregulation C) Myocardial dysfunction D) Minimal infl ammatory response Answer: C The post-cardiac arrest syndrome (PCAS) is an infl ammatory syndrome that best resembles sepsis. Infl ammatory mediators are released, resulting in activation of the coagulation cascade. Cerebral edema, ischemic degeneration, and impaired autoregulation characterize the brain injury pattern in the PCAS. Brain injury alone contributes greatly to overall morbidity and mortality in the resuscitated cardiac arrest patient. There is impaired autoregulation as well as impaired oxidative metabolism. There is predictable myocardial dysfunction. Myocardial dysfunction in the PCAS seems to be reversible and is characterized largely by global hypokinesis. Elevations of intracranial pressure are not prominent. Treatment during this period involves hemodynamic support and the use of inotropic and vasopressor agents if warranted. Hyperthermia should be avoided at all costs in patients with the PCAS. If aggressive therapy is pursued, consider sedation with hypothermia to improve neurological outcome in the ICU setting. A 72-year-old female is admitted with abdominal distension. She has history of colon cancer. Her last bowel movement was 4 days ago despite her taking scheduled polyethylene glycol. Her cancer was diagnosed 2 years ago and has been treated with chemotherapy after her disease was determined to be surgically unresectable. On physical exam the bowel is distended with absent bowel sounds. Lungs are normal. A nasogastric tube is placed with some mild improvement of distension. CT scan shows dilated loops of small bowel and colon with a transition point in the mid-descending colon. Which of the following will most likely improve this patient's ability to eat and ensure adequate caloric intake and fl uids? A) Referral for radiation B) Placement of a colonic stent across the single site of obstruction C) Fleet enema D) Exploratory surgery E) Placement of a venting percutaneous endoscopic gastrostomy (PEG) tube Answer: B A single-site bowel obstruction can be successfully palliated with colonic stent placement. Most self-expandable metal stent (SEMS) placement is a minimally invasive option for achieving acute colonic decompression in obstructed colorectal cancer. This would be a reasonable approach in this patient as opposed to surgery. When performed by experienced endoscopists, the technical success rate is high with a low procedural complication rate. Hand decontamination with either antisepticcontaining soaps, alcohol-based gels, or a combination has consistently been shown to reduce CLABSI rates. Skin antisepsis with chlorhexidine was found to be associated with a 50 % reduction in the subsequent risk of CLABSI compared with povidone iodine. Hypocalcemia has also been reported following massive transfusions due to the binding citrate agent. However, this is transient, and there is no evidence that calcium supplementation will be of benefi t. Septic shock and severe sepsis are also associated with hypocalcemia. This is due to abnormalities of vitamin D and parathyroid hormone. There is no evidence that septic patients benefi t from calcium repletion. The optimum dietary protein intake in patients with pressure ulcers is unknown, but may be much higher than the current adult recommendation of 0.8 g/kg/day. Increasing protein intake beyond 1.5 g/kg/day may not increase protein synthesis and may cause dehydration. It has been suggested that a reasonable protein requirement is therefore between 1.0 and 1.5 g/kg/day. Zinc and vitamin C are often included in supplements but have not been shown to improve healing in decubitus ulcers. Med. 1980; 133:485-92. 338 . A 35-year-old female is admitted with severe pain to her left foot. She states that she had a fracture of her ankle due to a fall 2 months ago. Since that time, she has had limited mobility and has infrequently gotten out of bed. She has had a follow-up appointment with her orthopedist who reports the ankle is healing well. She states that for the past 2 weeks, she has been completely unable to ambulate and has been bed bound. She reports a past medical history of anxiety and fi bromyalgia. On physical exam, the ankle is noted to be painful to mild touch. She states that the pain has a burning quality. The affected area is also noted to have an increased temperature, but no erythema is noted. X-rays are negative for fracture or any other noted pathology. What test would be most likely to make the diagnosis? A) Magnetic resonance imaging. B) Computed tomography C) Triple-phase bone scan D) Electromyography E) Depression screen Answer: C This patient's symptoms are consistent with a complex regional pain syndrome. This was formerly known as refl ex sympathetic dystrophy. This condition often occurs following trauma or surgery that results in a extended immobilization of the affected limb. Attempts have been made to quantify this syndrome. Criteria have been established to make the diagnosis. This includes pain due to mild stimuli and burning quality as well as changes in temperature, hair, and color of the affected extremity. Bone scan has been shown to reveal a typical pattern and can be a useful adjunct in confi rming the diagnosis. Diffuse increased perfusion to the entire extremity is usually noted. Therapy is directed toward nonnarcotic alternative medications that address neuropathic pain and increasing mobility to the affected area. Prevention focuses on early physical therapy. On exam, her temperature is 37.1 °C (98.8 °F), pulse rate is 90 beats per minute, respirations are 18 per minute, and blood pressure is 158/74 mmHg. Oxygen saturation by pulse oximetry is 96 %. The cardiopulmonary examination is normal. No edema is noted, but the left leg is shortened and externally rotated. Complete blood count and basic metabolic panel are normal. Chest radiograph is normal. Electrocardiogram shows sinus rhythm. Which of the following interventions is most likely to increase mortality in the postoperative period? A) Proceeding to surgery urgently in the next 48 h B) Prescribing a beta-adrenergic blocking agent within 24 h before surgery C) Postoperative venous thromboembolism prophylaxis D) Early postoperative mobilization E) Nicotine patch Answer: B A recent meta-analysis demonstrated that, despite a reduction in nonfatal myocardial infarction, perioperative betablockers started less than one day prior to noncardiac surgery were associated with an increased risk of death 30 days after surgery. Proceeding to surgery within 48 h has been shown to be benefi cial in hip fracture patients. Bouri S, Shun-Shin MJ, Cole GD, Mayet J, Francis DP. Metaanalysis of secure randomised controlled trials of betablockade to prevent perioperative death in non-cardiac surgery. Heart. 2014;100(6):456-64. 340. You are consulted to see a 36-year-old woman that has been admitted for shortness of breath to the obstetrics service. She is 4 months pregnant and has a prior history of asthma. She uses her albuterol inhaler several times per week to achieve symptomatic relief, but this has proven to be inadequate. History includes mild persistent asthma that was well controlled before her pregnancy with an as-needed short-acting β2-agonist and mediumdose inhaled glucocorticoids. On physical examination, vital signs are normal. The lungs have diffuse wheezes. She appears in minimal distress. Cardiac examination shows normal S1 and S2 with no gallops or murmurs. No leg edema is noted. What is the correct treatment? A) Prednisone. B) Add a long-acting β2-agonist. C) Add theophylline. D) Double the dose of inhaled glucocorticoid. E) A and B. Answer: E Approximately one-third of patients with asthma experience worsening of symptoms during pregnancy. Patients who present with mild exacerbations of asthma may be treated with bronchodilator therapy and steroids. Severe asthma exacerbations warrant intensive observation. Close monitoring of oxygen levels should be undertaken. Inhaled beta2-agonists are the mainstay of treatment. In particular, beta-adrenergic blocking agents should be avoided due to a possible increased bronchospastic effect. The early use of systemic steroids has not been shown to be detrimental and should be given when indicated. Intense follow-up care should occur. This may include referral to an asthma specialist. Reference Rey E, Boulet LP. Asthma in pregnancy. BMJ. 2007; 334(7593):582-5. 341 . A 32-year-old male is evaluated in the emergency department for diffuse muscle aches. He reports starting an extremely intense "boot camp" exercise routine 3 days ago. On physical examination, the patient is diffusely tender to touch. He appears uncomfortable. Arms and legs display moderate diffuse swelling. Temperature is normal, blood pressure is 92/50 mmHg, pulse rate is 120 beats/min, and respiratory rate is 20 breaths/min. Oxygen saturation is 97 %. Skin is mottled on the posterior back. Neurological examination fi ndings are nonfocal. Creatinine is 2.2 units/L, bicarbonate is 17 meq/L, and creatinine kinase (CPK) is 36,000 units/L. Which of the following is the most appropriate treatment for this patient? A) Hemodialysis B) Intravenous mannitol C) Rapid infusion of intravenous 0.9 % saline D) Rapid infusion of 5 % dextrose in water E) Surgical consultation Answer: C Rhabdomyolysis is a syndrome caused by extensive injury to skeletal muscle. It involves leakage of potentially toxic intracellular contents into plasma. This can occur in both the trained and non-trained athlete. This often occurs with the initiation of a new intense exercise regimen. The most severe complication is acute kidney injury (AKI). Etiologies of AKI may be related to hypovolemia, vasoconstriction, and myoglobin toxicity. Compartment syndrome of infl amed muscles may be either a complication of or the inciting cause of rhabdomyolysis. Mild diffuse swelling of muscle groups is common. Recommendations for the treatment of rhabdomyolysis include fl uid resuscitation fi rst and subsequent prevention of end-organ complications. This is best achieved with 0.9 % saline. Other measures to preserve kidney function may be considered after adequate volume has been given. Other supportive measures include correction of electrolyte imbalances. Fluids may be started at a rate of approximately 400 mL/h and then titrated to maintain a urine output of at least 200 mL/h. Treatment should continue until CPK displays a marked reduction or until the urine is negative for myoglobin. A 34-year-old woman is admitted overnight for the acute onset of pain after 10 days of bloody diarrhea. The diarrhea has escalated to 15 times per day. She has ulcerative colitis that was diagnosed 5 years ago. She currently takes azathioprine. On physical examination, she appears ill. Following aggressive fl uid resuscitation overnight, temperature is 38.6 °C (101.5 °F), blood pressure is 68/45 mmHg, pulse rate is 120 beats/min, and respiratory rate is 35 breaths/min. Abdominal examination discloses absent bowel sounds, distention, and diffuse marked tenderness with mild palpation. Radiographs on admissions reveal colonic distension of 5 cm. This am repeat radiographs reveal colonic distension of 8 cm. Which of the following is the most appropriate management? A) CT scan B) Immediate surgery C) Start infl iximab D) Start intravenous hydrocortisone E) Immediate gastroenterology consult Answer: B Early surgical consultation is essential for cases of toxic megacolon (TM). Indications for urgent operative intervention include free perforation, massive hemorrhage increasing toxicity, and progression of colonic dilatation which is the case here. Most guidelines recommend colectomy if persistent dilatation is present or if no improvement is observed on maximal medical therapy after 24-72 h. The rationale for early intervention is based on a marked increase in mortality after free perforation. The mortality rate for perforated, acute toxic colitis is approximately 20 %. Some recommend providing up to 7 days of medical therapy if the patient demonstrates clinical improvement despite persistent colonic dilatation. TM was fi rst thought to be the only complication of ulcerative colitis. It has been described in a number of conditions, including infl ammatory, ischemic, infectious, radiation, and pseudomembranous colitis. 345. An 85-year-old man with very poor functional status is admitted from the nursing home with severe shortness of breath. He has a history of a prior cerebrovascular accident that has resulted in right hemiparesis and aphasia. Chest X-ray shows that he has severe pneumonia. Before the entire family arrives, the patient is intubated immediately and transferred to the ICU. After a joint conference, the family decides to remove life support. Which of the following statements accurately characterizes ventilator withdrawal in this situation? A) You should suggest 24 more hours of observation. B) Limit family interaction while the patient is extubated. C) Pulse oximetry should be followed to help guide the family through the dying process. D) You should demonstrate that the patient is comfortable receiving a lower fraction of inspired oxygen (FIO2) before withdrawing the endotracheal tube. E) Such patients generally die within 30 min to an hour after the endotracheal tube is removed. Answer: D The family should be given the opportunity to be with the patient when the endotracheal tube is removed. The decision should be theirs to make and be a part of hospital protocol. All monitors including oxygen saturation should be turned off. The patient's comfort should guide therapy. FIO2 should be diminished to 20 %. The patient should be observed for respiratory distress before removing the endotracheal tube. Distress and air hunger can be treated with opioids and benzodiazepines prior to endotracheal tube removal. The family often expects an immediate response when the ventilator is turned off. It is important to inform them that the patient may live for hours to days. Also it is important to explain that you and staff will continue to follow and provide comfort during this period. End-of-life care is increasingly seen not as medical failure but a special time to assist the patient, family, and staff with the physical and emotional needs that occur with the dying of a patient. Resources, protocols, and education should be provided to staff to enhance these efforts. Answer: E The fat embolism syndrome typically presents 24-72 h after the initial injury. Dyspnea, tachypnea, and hypoxemia are the earliest fi ndings. This may progress to respiratory failure and a syndrome indistinguishable from acute respiratory distress syndrome (ARDS) may develop. Cerebral emboli produce neurological signs in up to 80 % of cases. This is often the second symptom to appear. The characteristic petechial rash may be the third component of the triad to occur. There is no specifi c therapy for fat embolism syndrome. Early immobilization of fractures has been shown to reduce the incidence of fat embolism syndrome and should be of primary importance with extensive long bone fractures. The risk is reduced by operative correction rather than conservative management. The use of steroids has been extensively studied for both prevention and treatment. It is recommended by some, for the management of the fat embolism syndrome. On admission amylase is 235 units/L, lipase is 175 unit/L, and alkaline phospatase is 52 g/dl. He is started in intravenous fl uids and has a rapid resolution of his symptoms the following day. Amylase on the second day is 38 units/L and lipase is 86 units/L. Ultrasound of the abdomen reveals a gallbladder with several stones. No gallbladder wall thickening is appreciated. What is the correct management of this patient? A) Discharge home with no further intervention. B) Surgical follow-up for cholecystectomy C) Cholecystectomy prior to discharge D) HIDA scan Answer: C If possible, patients admitted with gallstone pancreatitis should undergo cholecystectomy before discharge, rather than being scheduled as an outpatient. Patients discharged without a cholecystectomy are at high risk for recurrent bouts of pancreatitis. Recurrent episodes may be more severe than the original presentation. In one study, patients with mild gallstone pancreatitis who underwent laparoscopic cholecystectomy within 48 h of admission resulted in a shorter hospital stay. There was no apparent impact on the technical diffi culty of the procedure or the perioperative complication rate. 353. Which of the following will provide the best bowel preparation for a morning colonoscopy? A) 4 L polyethylene glycol-based preparation plus citric acid taken the evening before the procedure B) 2 L polyethylene glycol-based preparation taken the evening before the procedure C) 2 L of polyethylene glycol-based preparation on the evening before and 2 L of the same preparation on the morning of the procedure D) 1 L of polyethylene glycol-based preparation n the evening before and 1 L of the same preparation on the morning of the procedure Answer: C Signifi cant evidence exists that better colon preparation is associated with increased detection of colon polyps. Split-dose bowel preparation remains an essential concept for enhancing the quality of colonoscopy. This limits the amount of agent remaining in the colon prior to examination. Many bowel preparations for colonoscopy are available. No preparation has been shown to be superior to 4 L of a polyethylene glycol-based preparation split into two 2-L doses that are given the evening prior to and the morning of the procedure. A 67-year-old man with metastatic lung cancer is admitted for failure to thrive. During this admission, several end-of-life issues are addressed. He has chosen not to consider additional chemotherapy or radiation therapy. His cancer is unlikely to respond to such treatment. He and his family are focused on upcoming visits with his 4 children and 14 grandchildren over the next several weeks. However, the family reports that his lethargy, poor appetite, and depression will make this diffi cult. You estimate the patient's life expectancy to be weeks to several months. Which of the following would be the best management of this patient's symptoms? A) Initiation of a trial of a methylphenidate B) Referral of the patient to a psychologist C) Trial of a selective serotonin reuptake inhibitor D) Initiation of enteral feedings through a nasogastric tube E) Initiation of oral morphine Answer: A The use of psychostimulants, such as methylphenidate, is an effective management for cancer-related fatigue, opioidinduced sedation, and the symptoms of depression in the setting of a limited prognosis. Helping this patient achieve some of his end-of-life wishes is important. Psychostimulants have the benefi t of providing more immediate response than conventional therapies. It is improbable that this patient will live long enough to benefi t from cognitive behavioral therapy, SSRI, or nutritional support. Starting methylphenidate 2.5 mg PO BID is a reasonable choice when time is limited. Li M, Fitzgerald P, Rodin G. Evidence-based treatment of depression in patients with cancer. J Clin Oncol. 2012;30:1187-96. A 67-year-old man is admitted with severe right buttock pain. In the previous year, the patient underwent resection and laminectomy for metastatic renal cell tumor compressing his lower thoracic and upper lumbar spinal cord. The mass is inoperable, and he is receiving palliative chemotherapy. Hospice has not been discussed yet. During his admission, the pain has been severe and refractory to intravenous opioids. His daily requirement of hydromorphone is 150-175 mg for the past 4 days. On physical examination, vital signs are stable. He is somnolent, and when he wakes up he is in severe pain. Motor strength assessment is limited by pain. Which of the following should you recommend now? A) Trial of methylphenidate B) Placement of an implanted intrathecal drug pump C) Optimization of the opioid regimen D) A trial of intrathecal analgesia E) Lidocaine patch Answer: D This patient requires aggressive pain control measures. Changing opioid regimens will probably be of little benefi t. Evidence supports the use of intrathecal drug delivery systems compared with systemic analgesics in opioidrefractory patients. A trial of intrathecal medication is important, to determine the effect, prior to permanent placement of an implanted device. His previous laminectomy and associated scarring may limit the effect of intrathecal delivery as well as make catheter placement diffi cult. The use of palliative sedation therapy is indicated in patients with refractory symptoms at the end of life. Although his pain is severe and unresponsive to systemic medications, she is not at the end of life, nor have all interventions been pursued to address her pain. Deer TR, Smith HS, Burton AW et al. Comprehensive consensus based guidelines on intrathecal drug delivery systems in the treatment of pain caused by cancer pain. Pain Physician. 2011;14(3):E283-312. A 56-year-old woman has widely metastatic breast cancer. She is admitted for sepsis. The decision has been made to withdraw care and to allow a natural death preferably as an inpatient. The family is at the bedside. Oxygen saturation is 85 % with the patient receiving supplemental oxygen, 2 L/min by nasal cannula. On physical examination, she is nonverbal and restless in bed. Her respirations have become more difficult. The family appears fatigued and anxious. Which of the following should you do now? A) Request a sitter. B) Provide 100 % oxygen by face mask. C) Administer a dose of parenteral haloperidol. D) Administer a dose of parenteral morphine. E) Administer a dose of parenteral dexamethasone. Answer: D Morphine is the drug of choice with air hunger at the end of life. It is preferred over other sedation. There is no evidence that supplemental oxygen is benefi cial at the end of life. In addition, many patients experience increased agitation when a mask is placed over the mouth and nose. Family members may not desire a face mask for the patient as well during this special time. Ben-Aharon I, Gafter-Gvili A, Leibovici L, Stemmer SM. Interventions for alleviating cancer-related dyspnea: a systematic review and meta-analysis. Acta Oncol. 2012;51(8):996-1008. A 78-year-old woman who has recurrent breast cancer with metastasis is admitted for decreased appetite. Her last bowel movement was 4 days ago. She is on longacting morphine with oxycodone for breakthrough pain. Her bowel regimen is docusate, 100 mg twice daily. On physical examination, her abdomen is distended. A radiograph of the abdomen demonstrates a large amount of stool. She is given three enemas, which produce a small amount of stool. Which of the following is the most appropriate next step in the management of this patient's constipation? A) Administer lactulose. B) Administer methylnaltrexone. C) Administration of high-dose senna. D) Placement of a nasogastric tube (NGT) for highvolume laxative. E) Rotation to another opioid. Answer: B Methylnaltrexone is used for severe constipation in opioidinduced ileus. It is well tolerated in most instances. This patient has already shown an intolerance of stimulant laxatives; further measures are unlikely to be successful. An NGT would be uncomfortable. 358. A 68-year-old female is evaluated for preoperative clearance before she goes in for left knee elective surgery. She has a history of chronic hypertension. She has on amlodipine but has been noncompliant with her medicines. Her knee pain limits her activities but she is able to walk up two fl ights of stairs with minimal diffi culty. On physical exam her blood pressure is 145/99 mmHg, heart rate is 55 bpm, and respiratory rate is 11 breaths/ min. Extremities pulses are 2+ and bilateral. An echo done 7 months ago shows an ejection fraction of 30 %. The patient denies any new complaints. What is the next step? A) Proceed with surgery without additional preoperative testing. B) Control BP to ideal measurement of <130/85. C) Delay elective surgery for further evaluation or treatment. D) Exercise stress test. E) Start metoprolol. Answer: A Preoperative hypertension is frequently a hypertensive urgency, not an emergency. In general, patients with chronic hypertension may proceed to low-risk surgery as long as the diastolic BP is <110 mmHg. There continues to be some debate over the use of betablockers preoperatively. Current guidelines state that in patients with no risk factors, starting beta-blockers in the perioperative setting provides unknown benefi t. Thomas DR, Ritchie CS. Preoperative assessment of older adults. J Am Geriatr Soc. 1995;43 (7):811-21. 359. You are asked to admit a 32-year-old female for a 4-day history of lower abdominal pain that she describes as intermittent cramps. She denies nausea or vomiting. She also denies having urinary frequency, dysuria, and fl ank pain. Her only medication is an oral contraceptive agent. On physical examination, her temperature is 38.5 °C (101.4 °F), blood pressure is 120/68 mmHg, pulse rate is 100 beats/min, and respiratory rate is 18 breaths/min. Abdominal examination is normal. There is no fl ank tenderness. Pelvic examination shows cervical motion tenderness. Bilateral adnexal tenderness is appreciated on bimanual examination. She is in minimal distress and is tolerating liquids. The hematologic and serum chemistries are normal. Urine and serum pregnancy tests are negative. What is the next best step in the management of this patient? A) Consult for laparoscopic diagnosis and treatment. B) Admit the patient to the hospital, obtain pelvic ultrasound, and start ceftriaxone. C) Administer a single-dose IM ceftriaxone and discharge the patient. D) Administer a single-dose IM ceftriaxone and oral doxycycline for 14 days. E) Obtain pelvic and abdominal ultrasound and prescribe oral doxycycline with metronidazole. Answer: D This patient's clinical fi ndings are compatible with pelvic infl ammatory disease (PID). Women with mild to moderate PID may receive outpatient medical treatment without increased risk of long-term sequelae. Laparoscopy is the criterion standard for the diagnosis of PID, but the diagnosis of PID in emergency departments is often based on clinical criteria, without additional laboratory and imaging evidence. She should receive intramuscular ceftriaxone and oral doxycycline for 14 days. All women with suspected PID should be tested for infection with gonorrhea and chlamydia. In severe cases, imaging should be performed to exclude a tuboovarian abscess. Patients with PID should be hospitalized if there is (1) no clinical improvement after 48-72 h of antibiotics, (2) an inability to tolerate food or medicine, (3) severe symp-toms, (4) suspected abscess, (4) pregnancy, or (5) Answer: B One of the most common predisposing factors for erythema multiforme is infection with herpes simplex virus, which may or may not be active at the time of the EM eruption. EM is an acute, self-limited, and sometimes recurring skin condition that is considered to be a type IV hypersensitivity reaction. It is associated with infections, medications, and other various triggers. Patients with recurrent EM are typically treated with acyclovir or valacyclovir. Mycoplasma pneumonia, amoxicillin, ibuprofen, and cytomegalovirus may cause EM, but are not as common. Aurelian L, Ono F, Burnett J. Herpes simplex virus (HSV)associated erythema multiforme (HAEM): a viral disease with an autoimmune component. Dermatol Online J. 2003;9:1. A 65-year-old male with a long history of type II diabetes is admitted with the chief complaint of hematuria. His blood pressure is 130/65 mmHg. Otherwise his physical exam is normal. Urinalysis shows blood 3+ and protein 3+. No casts are seen. A 24-h urinary protein shows 8 g of protein and serum creatinine is normal. Urine microscopy shows isomorphic red blood cells with no casts. Renal and bladder ultrasound are normal. His hematuria is less by day 2 of his admission. What is the next most appropriate investigation? A) Renal angiogram B) Renal biopsy C) Doppler ultrasound of the kidneys D) CT scan of the abdomen and thorax alone E) Cystoscopy F) Observation alone 364. A 33-year-old woman is admitted to the hospital for evaluation of blurry vision and new-onset paraparesis. She has been followed closely by neurology in the past for two recent episodes of optic neuritis in the past 2 years. Her only other history is hypothyroidism. Her only medication is levothyroxine. On physical examination vital signs are normal. Visual acuity is 20/200 in the right eye and 20/30 in the left. Per ophthalmology consult, optic disks display pallor. Signifi cant spasticity is noted in her legs. The patient requires bilateral assistance to ambulate. Laboratory studies including a complete blood count, liver chemistry and renal function tests, and erythrocyte sedimentation rate are normal. The antinuclear antibody is positive. Anti-double-stranded DNA and anti-SSA/SSB antibodies are negative. Analysis of the cerebrospinal fl uid shows a normal IgG index and no abnormalities in oligoclonal banding. An MRI of the spinal cord reveals an increased signal extending over fi ve vertebral segments with patchy gadolinium enhancement. An MRI of the brain shows no abnormalities. Which of the following is the most appropriate next diagnostic test? A) Electromyography B) Serum antineutrophil cytoplasmic antibody test C) Serum neuromyelitis optica (NMO)-IgG autoantibody test D) Testing of visual evoked potentials E) Neuromyelitis optica (NMO)-IgG autoantibody test F) CSF to serum protein ratio Answer: E Neuromyelitis optica (NMO), the presentation of myelitis and optic neuritis, may be a variant of multiple sclerosis (MS) or a unique disease. This patient very likely has neuromyelitis optica (NMO). She should be tested for the autoantibody marker NMO-IgG. Differentiating between NMO and MS early in the disease may be important because the prognosis and treatment of the two diseases are different. NMO is a more severe disease treated with immunosuppressive drugs. MS is often initially treated with immunomodulatory therapies, such as β-interferon and glatiramer acetate. The MRI is suggested of NMO. In typical MS, lesions are usually less than two segments in length. The NMO-IgG test is approximately 75 % sensitive and more than 90 % specifi c for NMO. Cognitive trajectories after postoperative delirium Multifactorial index of cardiac risk in noncardiac surgical procedures Derivation and prospective validation of a simple index for prediction of cardiac risk of major noncardiac surgery Early surgery versus conventional treatment for infective endocarditis Management of infective endocarditis: challenges and perspectives Evidence that gabapentin reduces neuropathic pain by inhibiting the spinal release of glutamate Gabapentin for acute and chronic pain Constant observation in medicalsurgical settings: a multihospital study In their own time: the family experience during the process of withdrawal of life-sustaining therapy On day 3 she is started on tube feeds at 40 ml/h. Her goal rate is 70 ml h. Four hours after her tube feeds are started, gastric residuals are measured to be 375 ml. Which of the following should you recommend now? A) Withhold the feeding for 2 h C) Continuing the feeding at the current rate Advancing the feedings toward the patient's goal rate Poor validity of residual volumes as a marker for risk of aspiration in critically ill patients What is the best method for assessing pain in the nonverbal patient? E In nonverbal patients, pain assessment relies less on vitalsign changes and more on observing behaviors Fat embolism and the fat embolism syndrome 4-Liter split-dose polyethylene glycol is superior to other bowel preparations, based on systematic review and meta-analysis A predictive model identifi es patients most likely to have inadequate bowel preparation for colonoscopy High-dose methylprednisolone in the treatment of active ulcerative colitis Predicting outcome in severe ulcerative colitis On physical exam, he has moderate diffuse joint tenderness which is no different from his baseline. He has some nontender bumps palpated on the forearm bilaterally near to the olecranon process and displacement of metacarpal bones over the proximal phalanges with fl exion at proximal joints and with extension of distal interphalangeal joints. Labs are within normal range CT scan of the neck prior to surgery C) Avoidance of a paralytic drug during surgery D) Radiograph of the neck in fl exion and extension A serum autoantibody marker of neuromyelitis optica: distinction from multiple sclerosis The clinical course of neuromyelitis optica (Devic's syndrome) Answer: D In this patient, the feedings should be increased toward the goal rate. There is no correlation between gastric residual volume and the incidence of aspiration. Evidence shows that checking gastric residuals doesn't provide reliable information on tube-feeding tolerance, aspiration risk, or gastric emptying. Current guidelines recommend withholding feedings for gastric residual volumes greater than 500 mL.Answer: E This man has hematuria without evidence of dysmorphic red cells or casts in urinary sediment. Macroscopic hematuria in the absence of signifi cant proteinuria or RBC casts is an indication for imaging to exclude malignancy or cystic renal disease. Approximately 80-90 % of patients with bladder cancer present with painless gross hematuria. Urine cytology is extremely valuable but would not eliminate the need for cystoscopy, which is the standard for diagnosing bladder cancer. Many bleeding urinary tract lesions arise in the bladder and lower urinary tract, and no imaging technique is completely satisfactory for ruling out disease at these sites. Further imaging may be of use but cystoscopy will ultimately be needed. Answer: E The American College of Gastroenterology practice guidelines defi ne severe colitis as the passage of six or more stools per day with evidence of systemic toxicity. Intravenous corticosteroids, which are essential in severe cases, are effective in the induction of remission in the majority of cases. A daily intravenous steroid dose of hydrocortisone 300 mg or methylprednisolone 60 mg is suggested. Fortunately, most patients with severe UC respond to intravenous steroid therapy. However, 30 % of patients fail to respond after 5-7 days. These patients are considered to be steroid refractory. One of the simplest algorithms predicts that at the third day of intravenous steroid therapy, patients with a stool frequency of greater than eight per day or three per day plus a CRP greater than 45 mg/dl have an 85 % likelihood of requiring colectomy. Medical treatment of steroid-refractory severe UC has expanded with the availability of both cyclosporine and infl iximab as rescue agents. The need for colectomy may be reduced with the use of these agents. In addition, stool samples should be collected for culture and toxin analysis to rule out enteric infection.Answer: D Patients with -RA presenting for TKR represent those patients who have failed medical management and are a high-risk group for cervical spine involvement. Radiographic screening of RA patients presenting for joint replacement surgery reveals cervical spine instability in 44 %, which is typically asymptomatic. Lateral fl exion/extension views are more sensitive and are recommended. Cervical spine subluxation is less likely in RA patients presenting for general surgery, and there is currently no consensus on who should be screened in this population.